EMG Flashcards

1
Q

Electrophysiologic findings of compound muscle action potential conduction block and temporal dispersion, prolonged minimum F-wave latency, and reduced conduction velocity would most likely be seen in

a. Charcot-Marie-Tooth Disease.
b. myasthenic syndrome
c. Guillan-Barre syndrome
d. amyloidosis

A

c. All the findings mentioned are features associated with an acquired demyelinating condition such as Guillan-Barre syndrome or acute inflammatory demyelinating polyradiculoneuropathy (AIDP). Hereditary motor sensory neuropathies do not usually have temporal dispersion of compound muscle action potentials. Myasthenic syndrome is a neuromuscular junction disorder and amyloidosis is associated with a form of axonal peripheral neuropathy

How well did you know this?
1
Not at all
2
3
4
5
Perfectly
1
Q

You are treating a 48-year-old man who has two lumbar laminectomies for what you suspect is a recurrent right L5 radiculopathy. You perform an electromyogram to confirm the diagnosis, and it reveals 2+ positive waves and fibrillations with decreased recruitment in the right anterior tibialis. The patient informs you that he can only tolerate examination of one more muscle. Of the following you would choose:

A) extensor hallucis longus.
B) L5 paraspinals.
C) vastus medialis.
D) flexor digitorum longus

A

D) The history is suggestive of an L5 radiculopathy. Given the previous laminectomies, examining a single level of paraspinals would provide limited information. Although you cannot form any firm conclusions based on such a limited examination, study of the flexor digitorum longus will provide findings outside the peroneal, distribution and lend support to the clinical diagnosis.

How well did you know this?
1
Not at all
2
3
4
5
Perfectly
2
Q

Which technique may reduce stimulus artifact when performing sensory nerve conduction studies?

A) increasing the impedance of recording electrodes

b) increasing the stimulus duration
c) rotating the anode around the cathode

D) decreasing the low frequency filter

A

C) Rotating the anode around the cathode can decrease stimulus artifact. The other choices have no effect, or increase it

How well did you know this?
1
Not at all
2
3
4
5
Perfectly
3
Q
  1. Of the following, somatosensory studies would be the most useful in the diagnosis of
    (a) tarsal tunnel syndrome.
    (b) motor neuron disease.
    (c) myasthenia gravis.
    (d) multiple sclerosis.
A
  1. (d) Somatosensory studies can be helpful in the diagnosis of multiple sclerosis. Standard nerve
    conduction studies and electromyography are far more useful in the diagnosis of the other
    disorders.
How well did you know this?
1
Not at all
2
3
4
5
Perfectly
4
Q
  1. You are treating a 48-year-old man who has had two lumbar laminectomies for what you suspect is
    a recurrent right L5 radiculopathy. You perform an electromyogram to confirm the diagnosis, and
    it reveals 2+ positive waves and fibrillations with decreased recruitment in the right anterior
    tibialis. The patient informs you that he can only tolerate the examination of one more muscle. Of
    the following you would choose
    (a) extensor hallucis longus.
    (b) L5 paraspinals.
    (c) vastus medialis.
    (d) flexor digitorum longus.
A
  1. (d) The history is suggestive of an L5 radiculopathy. Given the previous laminectomies, examining a
    single level of paraspinals would provide limited information. Although you cannot form any firm
    conclusions based on such a limited examination, study of the flexor digitorum longus will provide
    findings outside the peroneal distribution and could lend support to the clinical diagnosis.
How well did you know this?
1
Not at all
2
3
4
5
Perfectly
5
Q
  1. The number of phases of a motor unit is related to the
    (a) conduction time through collateral nerve sprouts.
    (b) sweep speed setting on the oscilloscope screen.
    (c) sensitivity (gain) setting on the oscilloscope screen.
    (d) central conduction time and the state of the upper motor neuron.
A
  1. (a) The number of phases of the motor unit potential represents the synchronization of firing of the
    individual muscle fibers in a motor unit and is related to conduction time through collateral sprouts
    of the nerve. The number of phases is increased under conditions in which some sprouts are poorly
    myelinated and conduction is slow and less synchronous. The other factors do not affect the
    number of phases of a motor unit potential.
How well did you know this?
1
Not at all
2
3
4
5
Perfectly
6
Q
  1. During the electromyographic evaluation of a patient, you note discharges consistent with
    myotonia and small motor units in the distal muscles of the upper and lower extremities. The most
    likely diagnosis is
    (a) paramyotonia congenita.
    (b) myotonic dystrophy.
    (c) myotonia congenita.
    (d) hyperkalemic periodic paralysis.
A
  1. (b) Although myotonic discharges may be seen in all of these disorders, myotonic dystrophy is the
    only one with low-amplitude motor units in the distal muscles.
How well did you know this?
1
Not at all
2
3
4
5
Perfectly
7
Q
  1. Dorsal ulnar cutaneous nerve conduction studies are most useful in differentiating
    (a) ulnar neuropathy at the elbow from lower trunk plexopathy.
    (b) ulnar neuropathy at the elbow from ulnar neuropathy at the wrist.
    (c) lower trunk plexopathy from medial cord plexopathy.
    (d) ulnar neuropathy at the cubital tunnel from ulnar neuropathy at the ulnar groove.
A
  1. (b) The fibers of the dorsal ulnar cutaneous nerve travel in the lower trunk and the medial cord of the
    brachial plexus. This nerve travels with the ulnar nerve to the forearm, where it branches off
    proximal to the wrist and supplies sensation to the ulnar aspect of the dorsum of the hand and
    wrist. The dorsal ulnar cutaneous sensory nerve action potential amplitude could be decreased with
    a lesion of the lower trunk, the medial cord, or the ulnar nerve at the elbow or proximal forearm,
    and would not be useful in differentiating among them. It should be normal in ulnar neuropathy at
    the wrist.
How well did you know this?
1
Not at all
2
3
4
5
Perfectly
8
Q

  1. The potentials above are
    (a) complex repetitive discharges.
    (b) myotonic discharges.
    (c) neuromyotonia.
    (d) myokymia.
A
  1. (b) The potentials noted are single-fiber discharges waxing and waning in frequency and amplitude.
    This is characteristic of myotonic discharges.
How well did you know this?
1
Not at all
2
3
4
5
Perfectly
9
Q
  1. Comparing the results of electrodiagnostic studies on patients with clinical evidence of postpolio
    syndrome and the results of those obtained in persons with a history of polio of similar severity and
    duration since polio onset but no clinical evidence of postpolio syndrome, you find
    (a) smaller motor units in the symptomatic group.
    (b) more polyphasic motor units in the symptomatic group.
    (c) more fasciculations in the symptomatic group.
    (d) no significant differences between the groups.
A
  1. (d) There are no significant differences between the groups. Electrodiagnostic studies are not
    performed to confirm the diagnosis of postpolio syndrome; this is a clinical diagnosis. They are
    performed to rule out other disorders in the differential diagnosis.
How well did you know this?
1
Not at all
2
3
4
5
Perfectly
10
Q
  1. In testing a patient with suspected myasthenia gravis, needle electromyography (EMG) of the right
    upper extremity and orbicularis oculi is normal. Repetitive stimulation of the right ulnar nerve at a
    rate of 2/second shows no decrement before or immediately after 1 minute of exercise. A 4%
    decrement is noted 2 minutes after exercise. Your next electrodiagnostic step should be
    (a) EMG of the lower extremities.
    (b) EMG of the frontalis before and after edrophonium (Tensilon).
    (c) repetitive stimulation of recording from a proximal muscle.
    (d) ulnar somatosensory evoked potentials.
A
  1. (c) In patients with myasthenia gravis, repetitive nerve studies recorded from proximal muscles are
    more sensitive, though technically more difficult.
How well did you know this?
1
Not at all
2
3
4
5
Perfectly
11
Q
  1. In evaluating a hypotonic infant with electromyography you find low-amplitude, short-duration
    motor units with early recruitment. Based on these findings, the LEAST likely diagnosis would be
    (a) central core disease.
    (b) nemaline myopathy.
    (c) type II glycogenosis (acid maltase deficiency).
    (d) infantile spinal muscular atrophy.
A
  1. (d) The motor unit changes noted are typically seen in myopathies. Spinal muscular atrophy is an
    anterior horn cell disease.
How well did you know this?
1
Not at all
2
3
4
5
Perfectly
12
Q
  1. Which hypothesis does NOT explain a normal electromyograph (EMG) in a patient who has a
    lumbar radiculopathy?

(a) Involvement of only the sensory root
(b) Limited sampling of muscles
(c) Oxycodone taken prior to the study
(d) Timing of the study

A
  1. (c) Pain medication has no effect on EMG findings. All the other choices can be an explanation for a
    normal EMG in a patient who has a lumbar radiculopathy.
How well did you know this?
1
Not at all
2
3
4
5
Perfectly
13
Q
  1. A 27-year-old previously healthy woman awoke with severe right scapular and shoulder pain 4
    weeks ago. There is no history of trauma. She has no constitutional symptoms. Three weeks ago,
    her pain began resolving and scapular winging developed. Electromyography (EMG) reveals 3+
    positive waves and fibrillations with markedly decreased recruitment in the right serratus anterior.
    EMG of the right deltoid, biceps, pronator teres, abductor pollicis brevis, first dorsal interosseous,
    and cervical paraspinals is normal, as is EMG of the left serratus anterior. The most likely
    diagnosis is

(a) systemic lupus erythematosus.
(b) compression neuropathy of the dorsal scapular nerve.
(c) idiopathic brachial neuropathy (neuralgic amyotrophy).
(d) C5 radiculopathy due to cervical disc herniation.

A
  1. (c) This is a classic history for neuralgic amyotrophy or idiopathic brachial plexopathy involving the
    long thoracic nerve. In 30% of patients with neuralgic amyotrophy, EMG abnormalities can be
    found in the asymptomatic upper extremity; however, the absence of such findings does not
    obviously exclude the diagnosis. The findings are inconsistent with the other diagnoses.
How well did you know this?
1
Not at all
2
3
4
5
Perfectly
14
Q
  1. A 47-year-old soldier presents with left finger extensor weakness after repetitive wrist extension
    exercises at the gym. Motor nerve conduction studies were as follows:

Extensor Indicis
Nerve Stimulation Site Amplitude(mV) Conduction Velocity (m/s)
L. Radial mid-forearm 6.0
L. Radial elbow 2.0 60
L. Radial spiral groove 2.0 65
R. Radial elbow 5.8

This patient has

(a) radial neuropathy just distal to the spiral groove with axonotmesis.
(b) radial neuropathy just distal to the spiral groove with neurapraxia.
(c) posterior interosseous neuropathy with axonotmesis.
(d) posterior interosseous neuropathy with neurapraxia.

A
  1. (d) There is conduction block across the mid-forearm consistent with a posterior interosseous
    neuropathy with neurapraxia.
How well did you know this?
1
Not at all
2
3
4
5
Perfectly
15
Q
  1. A 50-year-old man complains of paresthesias of the right lateral 3 ½ digits and wrist pain. Nerve
    conduction studies for the right arm (norm in parentheses) are as follows:

Motor
Nerve Distal Latency (ms) Amplitude (mV) Conduction Velocity (m/s)
R. Median 5.3 (5) 48 (>45)
R. Ulnar 3.7 (5) 52 (>45) forearm
8.5 50 across the elbow

Sensory
Nerve Stimulation Site Peak Latency(ms) Amplitude (

A

  1. (d) There is slowing of the median motor distal latency and the median sensory latency across the
    wrist, findings consistent with carpal tunnel syndrome.
How well did you know this?
1
Not at all
2
3
4
5
Perfectly
16
Q
  1. Which technique may reduce stimulus artifact when performing sensory nerve conduction studies?

(a) Increasing the impedance of the recording electrodes
(b) Increasing the stimulus duration
(c) Rotating the anode around the cathode
(d) Decreasing the low frequency filter

A
  1. (c) Rotating the anode around the cathode can decrease stimulus artifact. The other choices have no
    effect, or increase it.
How well did you know this?
1
Not at all
2
3
4
5
Perfectly
17
Q
  1. The type of recruitment in this graph (gain: 500 uV/div, Sweep: 10 ms/div ) is most often seen in

(a) normals.
(b) neuropathy.
(c) myopathy.
(d) poor patient effort.

A
  1. (b) There is a single motor unit firing at approximately 20Hz without another motor unit coming in.
    This is an example of decreased recruitment, which may be seen in neuropathy. In myopathy one
    may see early recruitment of motor units. In patients who give submaximal effort there may be only
    1 motor unit seen on the screen, but the firing rate is less than 20Hz.
How well did you know this?
1
Not at all
2
3
4
5
Perfectly
18
Q
  1. Regarding the electrodiagnostic testing of a patient with definite myasthenia gravis, which statement
    is TRUE?

(a) An increment on repetitive stimulation at 1Hz of up to 25% is expected.
(b) A stimulation rate of 2-3Hz is most useful in demonstrating a decrement.
(c) An initial low amplitude compound motor action potential after a supramaximal stimulus is
expected.
(d) Motor unit variability is reflected by decreased jitter during single fiber EMG.

A
  1. (b) A 2 to 3Hz stimulation is optimal for demonstrating a decremental response. At this rate there is no
    build up of Ca++ concentration within the nerve terminal and the amount of acetylcholine in the
    readily available stores diminishes, making failure of some of the neuromuscular junctions possible
    in those patients with an already small safety factor. A decrement of up to 10% on 2 to 3Hz
    repetitive stimulation is considered normal. Small CMAPs initially are more suggestive of
    myasthenic (Lambert-Eaton) syndrome than of myasthenia gravis. Single fiber EMG reveals
    increased jitter and may reveal blocking.
How well did you know this?
1
Not at all
2
3
4
5
Perfectly
19
Q
  1. The potentials shown in this graph are

(a) fibrillations.
(b) myopathic motor units.
(c) end plate spikes.
(d) complex repetitive discharges.

A
  1. (c) The duration of these potentials is approximately 5ms, too short for a motor unit. The initial
    deflection is negative, distinguishing this potential as an end plate spike rather than a fibrillation.
How well did you know this?
1
Not at all
2
3
4
5
Perfectly
20
Q
  1. Which muscle is innervated by the peroneal division of the sciatic nerve?

(a) Adductor magnus–anterior part
(b) Piriformis
(c) Semimembranosus
(d) Biceps femoris- short head

A
  1. (d) The anterior part of adductor magnus is innervated by the obturator nerve. The piriformis receives
    its own branch off the lumbosacral plexus. The semimembranosus is innervated by the tibial
    division of the sciatic nerve. Only the short head of the biceps femoris is innervated by the peroneal
    division of the sciatic nerve.
How well did you know this?
1
Not at all
2
3
4
5
Perfectly
21
Q
  1. Surface electrodes for recording antidromic sural nerve conduction studies are best placed

(a) posterior to the medial malleolus.
(b) posterior to the lateral malleolus.
(c) anterior to the medial malleolus.
(d) anterior to the lateral malleolus.

A
  1. (b) The sural nerve travels posterior to the lateral malleolus and is best recorded over this area.
How well did you know this?
1
Not at all
2
3
4
5
Perfectly
22
Q
  1. A 40-year-old patient presents with weakness and sensory loss in the left arm after a motor vehicle
    accident. An EMG study 4 weeks after the injury shows the following results:

Muscle Positive Waves Fibrillations Fasciculations Recruitment
L. Deltoid 2+ 2+ 1+ mod decreased
L. Biceps 0 0 0 normal
L. Latissimus dorsi 2+ 1+ 1+ mild decreased
L. Triceps 2+ 2+ 1+ mod decreased
L. Pronator teres 0 0 0 normal
L. Abd pollicis brevis 0 0 0 normal
L. 1st dorsal interosseous 0 0 0 normal
L. Paraspinals 0 0 0

Based on these findings what is the cause of the patient’s weakness?

(a) C6 radiculopathy
(b) Upper trunk plexopathy
(c) Posterior cord plexopathy
(d) Lateral cord plexopathy

A
  1. (c) The abnormalities noted are in a posterior cord distribution.
How well did you know this?
1
Not at all
2
3
4
5
Perfectly
23
Q
  1. A 34-year-old pregnant woman with nocturnal paresthesias of the right lateral 3 digits and pain in
    the wrist and forearm is seen for electrodiagnostic studies. The studies (norms in parentheses) reveal
    the following results:
    Nerve Latency (ms) Amplitude (mV) Conduction Velocity (m/s)
    Motor studies
    R. Median – Wrist 4.2 (5)
    R. Median – Elbow 4 (>5) 45 (>45)
    R. Ulnar – Wrist 3.4 (5)
    R. Ulnar – Elbow 6 (>5) 55 (>45)
    L. Median – Wrist 3.9 (5)
    L. Median – Elbow 7 (>5) 52 (>45)
    L. Ulnar – Wrist 3.3 (5)
    L. Ulnar – Elbow 7 (>5) 56 (>45)

Muscle Positive Waves Fibrillations Recruitment
R. Deltoid 0 0 normal
R. Biceps 0 0 normal
R. Triceps 0 0 normal
R. Pronator teres 2+ 2+ decreased
R. Flex carpi radialis 2+ 2+ decreased
R. Flex carpi ulnaris 0 0 normal
R. Flex pollicis longus 2+ 2+ decreased
R. Abd pollicis brevis 2+ 2+ decreased
R. 1st Dorsal interosseous 0 0 normal
R. Cervical paraspinals 0 0
The patient’s symptoms are most likely due to an entrapment of the

(a) median nerve at the wrist (carpal tunnel syndrome).
(b) median nerve at the pronator teres muscle.
(c) median nerve at the ligament of Struthers.
(d) anterior interosseous nerve of the forearm.

A
  1. (c) An entrapment of the median nerve at the ligament of Struthers could involve all median innervated
    muscles of the forearm, including the pronator teres. Pronator teres syndrome usually does not
    involve the pronator teres since it is innervated from a branch of the median nerve that is more
    proximal. The patient does not have slowing of the median distal latencies suggestive of carpal
    tunnel syndrome, and the EMG abnormalities include abnormalities in more muscles than can be
    explained by an anterior interosseous neuropathy.
How well did you know this?
1
Not at all
2
3
4
5
Perfectly
24
Q
  1. Which radial innervated muscle is innervated by the C5 root?
    (a) Anconeus
    (b) Extensor carpi radialis longus
    (c) Brachioradialis
    (d) Triceps
A
  1. (c) None of the other muscles listed receives C5 innervation. The supinator is the only other radial
    innervated muscle that has C5 innervation.
How well did you know this?
1
Not at all
2
3
4
5
Perfectly
25
Q
  1. Which myopathy is most likely to demonstrate a “normal” EMG?
    (a) Myotonic dystrophy
    (b) Polymyositis
    (c) Steroid myopathy
    (d) Duchenne muscular dystrophy
A
  1. (c) In steroid myopathy the only abnormalities are atrophy of the type II fibers. Since these fibers are
    recruited last, when the screen is full of motor units, it is usually difficult to appreciate subtle
    amplitude changes. The other myopathies noted typically may exhibit positive waves and
    fibrillations with motor unit changes in the type I fibers.
How well did you know this?
1
Not at all
2
3
4
5
Perfectly
26
Q
  1. Which electrodiagnostic finding is more common in radiation plexopathy than in neoplastic
    plexopathy?
    (a) Myokymic discharges
    (b) Fibrillations
    (c) Decreased motor unit recruitment
    (d) Decreased amplitude of the sensory nerve action potential
A
  1. (a) Myokymia is present in 50% of patients who have radiation plexopathy, but is rarely seen in
    neoplastic plexopathy. The other findings noted in both plexopathies, but predominate in neither.
How well did you know this?
1
Not at all
2
3
4
5
Perfectly
27
Q
  1. Where do you place the cathode when performing a median motor nerve conduction study,
    stimulating at the elbow?
    (a) Lateral to the biceps tendon
    (b) Medial to the brachial artery
    (c) Lateral to brachioradialis
    (d) Medial to brachioradialis
A
  1. (b) The median nerve is just medial to the brachial artery at the elbow.
How well did you know this?
1
Not at all
2
3
4
5
Perfectly
28
Q
  1. What is the earliest electrophysiologic abnormality seen in generalized myasthenia gravis?
    (a) Increased jitter on single fiber electromyography of the extensor digitorum communis.
    (b) Blocking on single fiber electromyography of the extensor digitorum communis.
    (c) A 10% decrement of compound motor action potential (CMAP) amplitude with 2–3Hz
    repetitive stimulation, recording from the abductor digiti minimi.
    (d) A 10% decrement of CMAP amplitude with 2–3Hz repetitive stimulation, recording from the
    frontalis.
A
  1. (a) Increased jitter on single-fiber electromyography is the earliest abnormality seen in myasthenia
    gravis. The other abnormalities noted are seen later in the disease process.
How well did you know this?
1
Not at all
2
3
4
5
Perfectly
29
Q
  1. A 52-year-old woman is seen for follow-up 1 year after right carpal tunnel release. She had good
    initial relief of her symptoms following the release, but has had a 4-month history of recurrent
    symptoms in the right wrist and hand. Electromyography of the right upper extremity and cervical
    paraspinals is normal. Her nerve conduction studies (with normal values in parentheses) are as
    follows:
    Motor Nerve Conduction
    Nerve Segment Distal Latency (ms) Amplitude (mV) Velocity (m/s)
    R. Median forearm 4.5 (5) 50 (>45)
    R. Ulnar forearm 3.4 (5) 52 (>45)
    R. Ulnar across elbow 9.5 55
    L. Median forearm 3.5 10.0 (>5) 54 (>45)
    Sensory
    Nerve Segment Peak Latency (ms) Amplitude (uV)
    R. Median 14cm antidromic-digit II 3.9 (10)
    R. Median 7cm antidromic-digit II 1.9 15
    R. Ulnar 14cm antidromic-digit V 3.2 (10)
    You conclude that the patient has
    (a) compression of the median nerve at Guyon’s canal.
    (b) pronator syndrome.
    (c) entrapment of the median nerve at the ligament of Struthers.
    (d) normal postoperative findings.
A
  1. (d) After a successful carpal tunnel release median distal latencies improve, but often do not return to
    normal. Mild residual slowing is not unusual.
How well did you know this?
1
Not at all
2
3
4
5
Perfectly
30
Q
  1. Which potentials have manifestations that CANNOT be observed on physical examination?
    (a) Fasciculation potentials
    (b) Myokymic discharges
    (c) Complex repetitive discharges
    (d) Cramp potentials
A
  1. (c) Complex repetitive discharges can only be detected with electromyography. The remainder of the
    discharges have manifestations that can be observed on physical examination.
How well did you know this?
1
Not at all
2
3
4
5
Perfectly
31
Q
  1. Which statement regarding myotonic discharges is TRUE?
    (a) They are variable in frequency.
    (b) They have a constant amplitude.
    (c) They arise from the anterior horn cell.
    (d) They are rarely seen without clinical myotonia.
A
  1. (a) Myotonic discharges consist of repetitive waveforms of similar configuration which wax and wane
    in their frequency and amplitude.
How well did you know this?
1
Not at all
2
3
4
5
Perfectly
32
Q
  1. A medical student has been practicing with the electromyography (EMG) machine. You note that
    the conventional filter settings have been changed. The low frequency filter is now set at 200Hz and
    the high frequency filter is set at 1,000Hz. This will cause
    (a) a decreased common mode rejection ratio.
    (b) increased input impedance.
    (c) no significant changes.
    (d) distortion of recorded potentials.
A
  1. (d) Typical settings are 20Hz for the low frequency filter and 10,000Hz for the high frequency filter.
    Allowing such a narrow bandwidth will cause distortion of the recorded potentials. It would have
    no effect on the input impedance or common mode rejection ratio.
How well did you know this?
1
Not at all
2
3
4
5
Perfectly
33
Q
  1. In a patient with early generalized myasthenia gravis which electromyographic abnormality is the
    most likely?
    (a) Fibrillation potentials
    (b) Fasciculation potentials
    (c) Amplitude variability in a single motor unit
    (d) Myokymic discharges
A
  1. (c) This can be noted using a trigger and delay line and observing a single motor unit firing
    repetitively. The other abnormalities are not seen in myasthenia gravis.
How well did you know this?
1
Not at all
2
3
4
5
Perfectly
34
Q
  1. A 27-year-old woman noted the onset of paresthesias in the lateral 3 digits of the right hand 6
    months ago. Initially, these symptoms were constant, lasting for about 1 week, and then resolved
    spontaneously. Over the 3 weeks prior to this consultation the paresthesias recurred and they are
    now present in both the right upper and lower extremities. Electromyogram of the right upper
    extremity is normal. Bilateral median, ulnar, and sural sensory nerve conduction studies were
    normal. Right peroneal motor nerve conduction study was normal. What electrodiagnostic study
    would be most appropriate to perform at this time?
    (a) Single-fiber electromyography
    (b) Somatosensory evoked potentials
    (c) Right median F wave
    (d) Electromyogram of the right lower extremity
A
  1. (b) With all of the studies being normal a diagnosis of multiple sclerosis should be considered. Of
    those listed the only study that might provide any useful information would be the somatosensory
    evoked potentials.
How well did you know this?
1
Not at all
2
3
4
5
Perfectly
35
Q
  1. Five days after a motorcycle accident a 25-year-old man presents with an insensate right thumb and
    index finger and right upper extremity weakness. Electrodiagnostic studies (with normal values in
    parentheses) are as follows:
    Nerve Conduction Studies
    Motor
    Nerve Distal Latency (ms) Amplitude (mV) Conduction Velocity (m/s)
    R. Median 3.8 (5) 50 (>45)
    R. Ulnar 3.5 (5) 55 (>45)
    L. Median 3.4 (5) 54 (>45)
    Sensory
    Nerve Peak Latency (ms) Amplitude (uV)
    R. Median (index) 3.6 (10)
    R. Ulnar (little) 3.4 (10)
    L. Median (index) 3.5 (10)
    EMG
    Muscle Fibrillations Positive Waves Motor Unit Size Recruitment
    R. Deltoid 0 0 Normal Mod Decreased
    R. Biceps 0 0 Normal Mod Decreased
    R. Pronator Teres 0 0 Normal Mod Decreased
    R. Triceps 0 0 Normal Normal
    R. First dorsal interosseous 0 0 Normal Normal
    R. Abductor pollicis brevis 0 0 Normal Normal
    R. Paraspinals
    C2-4 0 0
    C5-7 0 0
    C8 0 0
    What is the most likely diagnosis?
    (a) C6 radiculopathy
    (b) Upper trunk brachial plexopathy
    (c) Lateral cord brachial plexopathy
    (d) No conclusions, since the studies were performed too early
A
  1. (a) The presence of an insensate thumb and index finger with a normal median sensory nerve action
    potential (SNAP) recorded from the index finger is indicative of a lesion proximal to the dorsal root
    ganglion, such as root avulsion. Positive waves in the paraspinals are usually seen 1 week post
    onset. While the full EMG findings may not be apparent for 3 to 4 weeks, the data presented are
    sufficient to draw conclusions.
How well did you know this?
1
Not at all
2
3
4
5
Perfectly
36
Q
  1. Studying F-wave responses would be most helpful in making an early diagnosis of
    (a) cervical radiculopathy.
    (b) myotonic dystrophy.
    (c) multiple sclerosis.
    (d) Guillain-Barré syndrome.
A
  1. (d) Guillain-Barré syndrome (acute idiopathic demyelinating polyradiculoneuropathy) commonly
    affects the most proximal portion of the nerve while sparing the main nerve trunk in the early
    stages. While motor conduction velocities obtained with ordinary techniques may be normal, F-
    wave studies help detect proximal abnormalities.
How well did you know this?
1
Not at all
2
3
4
5
Perfectly
37
Q
  1. In electromyographic testing of the ulnar nerve for entrapment at the elbow, the primary reason for
    flexing the elbow to 45° or more is that this position
    (a) causes the ulnar nerve to be more superficial and thus easier to stimulate.
    (b) allows more accurate assessment of the length of the nerve.
    (c) allows for the least possible stimulus overflow to the radial nerve.
    (d) causes increased compression of the nerve, thus improving the diagnostic yield.
A
  1. (b) For testing of ulnar conduction around the elbow, flexion of 45° or greater is preferred because
    conduction studies frequently show slowing in the elbow segment when the fully extended position
    is used. The discrepancy is thought to arise from the underestimation of the actual length of the
    nerve when using surface distance measurement.
How well did you know this?
1
Not at all
2
3
4
5
Perfectly
38
Q
  1. Which of the following is an axonal peripheral neuropathy?
    (a) Hereditary motor and sensory neuropathy type I
    (b) Diphtheria neuropathy
    (c) Metachromatic leukodystrophy
    (d) Vincristine neuropathy
A
  1. (d) Vincristine can cause an axonal peripheral neuropathy. The others listed are demyelinating
    peripheral neuropathies.
How well did you know this?
1
Not at all
2
3
4
5
Perfectly
39
Q
  1. Electrophysiologic findings of compound muscle action potential conduction block and temporal
    dispersion, prolonged minimum F-wave latency, and reduced conduction velocity would most
    likely be seen in
    (a) Charcot-Marie-Tooth disease.
    (b) myasthenic syndrome.
    (c) Guillain-Barré syndrome.
    (d) amyloidosis.
A
  1. (c) All the findings mentioned are features associated with an acquired demyelinating condition such as Guillain-Barré syndrome or acute inflammatory demyelinating polyradiculoneuropathy (AIDP).
    Hereditary motor sensory neuropathies do not usually have temporal dispersion of compound
    muscle action potentials. Myasthenic syndrome is a neuromuscular junction disorder and
    amyloidosis is associated with a form of axonal peripheral neuropathy.
How well did you know this?
1
Not at all
2
3
4
5
Perfectly
40
Q
  1. Which filter setting is usually considered to be appropriate for routine needle electromyography?
    Low frequency High frequency
    (a) 2–10Hz 10,000Hz
    (b) 2–10Hz 2,000Hz
    (c) 20–30Hz 10,000Hz
    (d) 20–30Hz 2,000Hz
A
  1. (c) There is no universally accepted guideline for filter settings. However, based on clinical experience
    certain ranges have been determined and are recommended. Each procedure has particular filter
    settings, which are based on optimum frequency content of mean waveforms routinely observed.
    The recommended filter setting for routine needle electromyography is 20–30 hertz for the low
    filter and 10kilo hertz for the high filter.
How well did you know this?
1
Not at all
2
3
4
5
Perfectly
41
Q
  1. A 35-year-old woman is sent to you for electrodiagnostic evaluation. Her symptoms include
    numbness in the right 4th and 5th digits and in the medial hand distal to the wrist. Your primary
    diagnosis is MOST likely to be
    (a) ulnar neuropathy.
    (b) C8/T1 radiculopathy.
    (c) lower trunk plexopathy.
    (d) medial cord plexopathy.
A
  1. (a) The distribution described is most consistent with involvement of the ulnar nerve proximal to the
    dorsal cutaneous branch. Involvement of the C8/T1 roots, lower trunk of the brachial plexus, or
    medial cord of the brachial plexus would usually result in some additional sensory deficits over the
    medial forearm proximal to the wrist.
How well did you know this?
1
Not at all
2
3
4
5
Perfectly
42
Q
  1. The potential above is
    (a) a fibrillation.
    (b) an end plate spike.
    (c) a voluntary motor unit.
    (d) a fasciculation.
A
  1. (d) The potential is of a duration consistent with a motor unit and fires only once over a 500-ms period. The firing rate would be inconsistent with a voluntary motor unit but is typical of a fasciculation.
    The amplitude and duration of the potential would also preclude an end plate spike or fibrillation
    potential. Furthermore, a fibrillation potential would have an initial positive deflection.
How well did you know this?
1
Not at all
2
3
4
5
Perfectly
43
Q
  1. Which statement is TRUE about volume conduction as it relates to electrophysiologic studies?

(a) Volume conduction is the transmission of an electrical potential through intracellular tissue.
(b) Volume conducted potentials produce a biphasic waveform as an advancing action potential
approaches and then passes beneath a recording electrode.
(c) The negative peak of a recorded waveform represents the time that the action potential is
beneath the active electrode.
(d) Volume conducted near field potential amplitudes does not characteristically depend on the
distance between the recording electrode and the source.

A
  1. (c) The characteristics of volume conducted near field responses are dependent on the distance from
    the recording electrode and the electrical source. The responses represent intracellular events
    transmitted through extracellular fluid and tissue. They usually produce a triphasic waveform and
    the negative phase is that time during which the advancing wave is directly underneath the
    recording electrode.
How well did you know this?
1
Not at all
2
3
4
5
Perfectly
44
Q
  1. Which statement is TRUE about volume conduction as it relates to electrophysiologic studies?

(a) Volume conduction is the transmission of an electrical potential through intracellular tissue.
(b) Volume conducted potentials produce a biphasic waveform as an advancing action potential
approaches and then passes beneath a recording electrode.
(c) The negative peak of a recorded waveform represents the time that the action potential is
beneath the active electrode.
(d) Volume conducted near field potential amplitudes does not characteristically depend on the
distance between the recording electrode and the source.

A
  1. (c) The characteristics of volume conducted near field responses are dependent on the distance from
    the recording electrode and the electrical source. The responses represent intracellular events
    transmitted through extracellular fluid and tissue. They usually produce a triphasic waveform and
    the negative phase is that time during which the advancing wave is directly underneath the
    recording electrode.
How well did you know this?
1
Not at all
2
3
4
5
Perfectly
45
Q
  1. Which statement is TRUE about F-wave and H-reflex responses?

(a) They both are obtained with supramaximal stimulation.
(b) F waves can only be obtained from posterior tibial and median nerves.
(c) They both involve conduction along motor and sensory fibers.
(d) They both can be helpful in the diagnosis of S1 radiculopathy.

A
  1. (d) The F-wave response is a pure motor response that is obtained with supramaximal stimulation and
    can be obtained from any motor nerve in an adult. The H reflex is usually obtained with
    submaximal stimulation and involves both sensory and motor fibers. Tibial and peroneal nerve F
    waves can be abnormal in L5–S1 radiculopathies and tibial nerve H reflex can be abnormal in S1
    radiculopathies.
How well did you know this?
1
Not at all
2
3
4
5
Perfectly
46
Q
  1. Which compound muscle action potential (CMAP) finding is the most compatible with myasthenia
    gravis?

(a) A 15% decrement in the amplitude between the 1st and 4th responses with repetitive
stimulation at 3 hertz
(b) A low amplitude response with supramaximal stimulation of the ulnar nerve and pick-up over
the hypothenar muscles
(c) A 100% increase in the amplitude with stimulation of the spinal accessory nerve and pick-up
over the trapezius muscle immediately post exercise
(d) A 50% increment in the amplitude between the 1st and 4th responses with repetitive
stimulation at 30 hertz

A

  1. (a) In myasthenia gravis there is a decrement in the amplitude and area of the compound muscle action
    potential (CMAP) with stimulation at low frequencies (2–3Hz). Stimulation at high rates of 20–
    50Hz can result in pseudofacilitation with correction of the amplitude decrement in normal subjects
    as well as patients with neuromuscular junction disorders. A 100% or greater increase in the
    amplitude of the post exercise CMAP is usually associated with a presynaptic neuromuscular
    junction disorder such as Lambert-Eaton myasthenic syndrome (LEMS). Small CMAPs on routine
    nerve conduction studies are more characteristic of LEMS than of myasthenia gravis.
How well did you know this?
1
Not at all
2
3
4
5
Perfectly
47
Q
  1. Which filter setting is usually considered appropriate for routine sensory nerve conduction studies?

LOW-FREQUENCY FILTER HIGH-FREQUENCY FILTER

(a) 2–10Hz 10,000Hz
(b) 2–10Hz 2,000Hz
(c) 20–30Hz 10,000Hz
(d) 20–30Hz 2,000Hz

A
  1. (b) There is no universally accepted guideline for filter settings. However, based on clinical
    experience, certain ranges have been determined and are recommended. Each procedure has
    particular filter settings based on the optimum frequency content of the mean waveforms that are
    routinely observed. The recommended filter setting for routine sensory nerve conduction studies is
    2–10Hz for the low-frequency filter and 2,000Hz for the high-frequency filter.
How well did you know this?
1
Not at all
2
3
4
5
Perfectly
48
Q
  1. The inferior gluteal nerve innervates which muscle?

(a) Gluteus maximus
(b) Gluteus medius
(c) Gluteus minimus
(d) Tensor fascia latae

A
  1. (a) The inferior gluteal nerve innervates the gluteus maximus muscle. The other 3 muscles are all
    innervated by the superior gluteal nerve.
How well did you know this?
1
Not at all
2
3
4
5
Perfectly
49
Q
  1. A 55-year-old man has had worsening symptoms of right arm pain and numbness in the right index
    and long fingers. He does not have any bowel or bladder problems or symptoms in the other limbs.
    His electrodiagnostic studies on the right (R) side show the following:

MOTOR NERVE CONDUCTION STUDIES
Nerve Stimulation Site Distal Latency (ms) Amplitude (mV) NCV (m/s)
R Median Wrist 3.6 8.4
Elbow 8.1 53
R Ulnar Wrist 3.5 8.9
Below elbow 8.5 57
Above elbow 8.1 55

SENSORY NERVE CONDUCTION STUDIES
Nerve Stimulation Site Recording Site Distal Latency (ms) Amplitude (µV)
R Median Wrist - 14 cm Digit II 3.2 55
R Ulnar Wrist - 14 cm Digit V 3.1 47
R Median- mixed Palm - 8 cm Wrist 1.9 76
R Ulnar - mixed Palm - 8 cm Wrist 1.8 45

NEEDLE ELECTROMYOGRAPHY

Muscle Abnormal Recruitment
Spontaneous
Activity
R Triceps 1+ Reduced
R Biceps 0 Normal
R Brachioradialis 1+ Reduced
R Extensor Digitorum
Communis 0 Normal
R Pronator Teres 2+ Reduced
R Flexor carpi ulnaris 0 Normal
R 1st dorsal interosseous 0 Normal
R Supraspinatus 0 Normal
R Cervical-paraspinals upper 0
R Cervical-paraspinals middle 2+
R Cervical-paraspinals lower 0

These findings are most consistent with right

(a) median neuropathy.
(b) radial neuropathy at the spiral groove.
(c) middle trunk plexopathy.
(d) C6/C7 radiculopathy.

A
  1. (d) The nerve conduction studies are normal. The needle electromyographic findings are limited to the muscles innervated by the C6 and C7 roots. Spontaneous activity in the cervical paraspinals makes
    a more distal lesion unlikely.
How well did you know this?
1
Not at all
2
3
4
5
Perfectly
50
Q
  1. A 73-year-old hospitalized patient complains of pain in the inguinal area 6 hours after a cardiac
    angiogram. Examination reveals hip flexion and knee extension weakness. There is decreased
    sensation over the medial ankle. Which test would you order first?

(a) Electrodiagnostic testing
(b) Arteriogram of the legs
(c) Venogram of the legs
(d) Computerized tomography of the pelvis

A
  1. (d) The nerve most likely affected in this type of situation is the femoral nerve, due to a retroperitoneal hematoma. The best way to assess for this acutely is by radiologic studies. Electrodiagnostic testing would be inappropriate in an acute setting, since needle examination findings usually take a few days to weeks to evolve, and the findings would not necessarily help in the acute management of this patient. Vascular studies may help identify the source of bleeding if the symptoms persist.
How well did you know this?
1
Not at all
2
3
4
5
Perfectly
51
Q
  1. Which one of the following is associated with an axonal loss sensory polyneuropathy?

(a) Lead poisoning
(b) Cisplatin chemotherapy
(c) Polyarteritis nodosa
(d) Charcot-Marie-Tooth disease (type 1)

A
  1. (b) Cisplatin is associated with an axonal loss sensory neuropathy. Lead usually causes upper limb
    weakness and patients usually have few or no sensory complaints. Polyarteritis nodosa is the most
    common of the necrotizing vasculitides and the most common pattern of nerve involvement is that
    of mononeuropathy multiplex. In Charcot-Marie-Tooth disease type 1 the primary pathology
    involves uniform demyelination of the peripheral nerves.
How well did you know this?
1
Not at all
2
3
4
5
Perfectly
52
Q
  1. What test is most sensitive for diagnosing myasthenia gravis?

(a) Facial nerve repetitive studies at 30 hertz
(b) Ulnar nerve repetitive studies at 3 hertz
(c) Single fiber electromyography
(d) Acetylcholine receptor antibodies

A
  1. (c) With a sensitivity of 92% to 100%, single fiber electromyography, which includes measurement of
    jitter, is the most sensitive test in assessing for myasthenia gravis. The sensitivity of repetitive
    stimulation of distal and proximal nerves is 77% to 100%, and acetylcholine receptor antibody
    sensitivity is 73% to 90%.
How well did you know this?
1
Not at all
2
3
4
5
Perfectly
53
Q
  1. A 23-year-old man is sent to the electrophysiology laboratory for evaluation of bilateral foot drop.
    His history is significant for recent treatment of Hodgkin’s lymphoma with surgery and
    chemotherapy. Which electrophysiologic finding is most consistent with a polyneuropathy
    secondary to vincristine use?

(a) Reduced conduction velocity of the motor nerves
(b) Absent sensory responses with preserved motor responses
(c) Small amplitude motor and sensory responses
(d) Motor and sensory conduction block

A
  1. (c) Vincristine causes an axonal polyneuropathy that primarily affects the most distal aspects of the
    nerve and produces a sensorimotor polyneuropathy. Nerve conduction studies reveal small
    amplitude or absent motor and sensory responses.
How well did you know this?
1
Not at all
2
3
4
5
Perfectly
54
Q
  1. A 10-year-old boy suffered a blunt injury to his arm with resultant wrist drop. Electrophysiologic
    studies done on the day of injury show normal nerve conduction studies in the nerves in the affected
    limb except for the radial nerve. The radial nerve responses are normal distally but with stimulation
    proximal to the site of injury responses are absent. Which finding is most consistent with a good
    prognosis for recovery?

(a) Normal radial motor response with stimulation distal to the injury on day 5
(b) Normal radial sensory response with stimulation distal to the injury on day 5
(c) Absence of fibrillation potentials in the radial innervated muscles on day 5
(d) Presence of fibrillation potential in the radial innervated muscles on day 15

A
  1. (a) It is difficult to differentiate between neurapraxia (good prognosis for recovery) and axonotemesis
    (poorer prognosis for complete recovery) immediately postinjury using electrophysiologic testing.
    The compound muscle action potentials (CMAPs) are normal with stimulation distal to the lesion
    initially for the first 2 to 3 days. In lesions involving the axons, the CMAPs drop significantly,
    reaching a nadir at about day 7. In contrast, the CMAPs remain the same size with distal stimulation
    in lesions that only produce conduction block. Similar changes are noted in the amplitudes of the
    sensory nerve action potentials (SNAPs); however, the amplitudes are unaffected for the first 5
    days, and then progressively fall over the next 4 to 5 days. If axonal damage exists, the needle
    electrode examination usually does not show fibrillation potentials until 2 to 3 weeks after the
    injury.
How well did you know this?
1
Not at all
2
3
4
5
Perfectly
55
Q
  1. A patient with a history of cervical cancer treated with pelvic radiation is sent to the
    electrodiagnostic laboratory for assessment of a suspected lumbosacral plexopathy (LSP). Which
    feature would more likely be seen in recurrent neoplasm than radiation plexopathy as the cause of
    the LSP?

(a) Membrane instability shown by needle electrode examination
(b) Weakness in the distal muscles of the affected limb
(c) Decreased amplitude of the sensory nerve action potentials
(d) Deep aching pain locally presenting within 3 months of treatment

A
  1. (d) Neoplasm is a common cause of lumbosacral plexopathy (LSP). Usually it is from direct extension of tumor from nearby regions. Patients with tumor extension usually present initially with pain,
    both locally and in a radicular pattern. Radiation induced LSP usually results in proximal weakness
    without any pain or with little pain and most commonly present years after treatment is completed.
    Membrane instability, distal weakness, and decreased motor and sensory responses can be seen
    both in tumor recurrence and in radiation induced LSP.
How well did you know this?
1
Not at all
2
3
4
5
Perfectly
56
Q
  1. A 45-year-old woman presents with a 2-week history of left leg weakness and paresthesias in the left
    leg for about 1 month. Examination reveals 4/5 strength with left foot dorsiflexion and plantar
    flexion, and 4+/5 strength with left knee flexion. Strength otherwise is 5/5 in the left lower
    extremity. Her nerve conduction and needle examination findings are as follows.
    MOTOR NERVE CONDUCTION STUDIES
    Nerve Stimulation Site Distal Latency (ms) Amplitude (mV) NCV (m/s)
    L Peroneal Ankle 4.5 4.4
    Below knee 4.1 48
    Above knee 3.8 51
    R Peroneal Ankle 4.2 5.9
    Below knee 5.5 52
    Above knee 5.1 55
    L Tibial Ankle 4.4 7.0
    Knee 6.5 53
    R Tibial Ankle 4.6 8.6
    Knee 8.1 55

SENSORY NERVE CONDUCTION STUDIES
Nerve Stimulation Site Distal Latency (ms) Amplitude (µV)
L Sural Ankle - 14 cm 3.6 8
R Sural Ankle - 14 cm 3.7 17
L Superficial peroneal Ankle - 12 cm 3.2 6
R Superficial peroneal Ankle - 12 cm 3.1 15

NEEDLE ELECTROMYOGRAPHY

Muscle Abnormal Recruitment
Spontaneous
Activity
Adductor longus 0 Normal
Quadriceps femoris 0 Normal
Iliopsoas 0 Normal
Semimembranosus 1+ Reduced
Biceps femoris (shorthead) 1+ Reduced
Tibialis anterior 2+ Reduced
Extensor hallucis longus 2+ Reduced
Medial gastrocnemius 2+ Reduced
Tibialis posterior 2+ Reduced
bTensor fascia latae 0 Normal
Gluteus maximus 0 Normal
Lumbar paraspinals 0

These findings are most consistent with a lesion located

(a) in the spine.
(b) between the spine and hip.
(c) between the hip and distal thigh.
(d) between the distal thigh and ankle.

A
  1. (c) The lesion is in the sciatic nerve affecting both the tibial and peroneal nerves. The lesion is distal to the origin of the gluteal nerves. The femoral nerve is not involved. The left lower extremity motor and sensory studies are normal but compared to the right side reveal small amplitude responses.
How well did you know this?
1
Not at all
2
3
4
5
Perfectly
57
Q
  1. A 40-year-old man sustained an injury to his left arm, 3 weeks ago, when he lost his balance and
    crashed into a bookshelf. His complaints include left arm pain, weakness with extension of his
    wrist and fingers, and decreased hand grip. He denies any numbness but has odd sensations over the
    dorsum of the left hand. Prior to any testing, which problem would you consider as the most
    likely?

(a) Posterior interosseous neuropathy
(b) C7 radiculopathy
(c) Posterior cord brachial plexopathy
(d) Radial neuropathy

A
  1. (d) Based on the clinical presentation, radial nerve injury is the most likely cause of the patient’s
    symptoms. Considering the location of the trauma the other possibilities seem less likely. In a
    posterior interosseous nerve injury one would not expect any sensory problems.
How well did you know this?
1
Not at all
2
3
4
5
Perfectly
58
Q
  1. Which description best localizes the extensor indicis proprius muscle (with the forearm fully
    pronated) for needle electrode examination?

(a) Junction of the upper and middle third of the forearm between the radius and ulna
(b) Four fingerbreadths proximal to the wrist and directly over the ulnar side of the radius
(c) Two fingerbreadths proximal to the ulnar styloid and just radial to the ulna
(d) Mid-forearm along the radial border of the ulna

A
  1. (c) Answer (a) describes the location of the extensor digitorum communis muscle; answer (b) describes the location of the extensor pollicis brevis muscle; and answer (d) describes the location of the
    extensor pollicis longus.
59
Q
  1. Blink reflex studies can be useful in diagnosing which condition?

(a) Neuromuscular junction disorder
(b) Axonal neuropathy
(c) Motor neuron disease
(d) Midpontine lesion

A
  1. (d) Blink reflex studies can help assess facial and trigeminal nerve lesions, as well as central lesions in the brain stem. Neuromuscular junction disorders are better assessed by repetitive studies. Axonal
    neuropathies rarely affect the blink reflex, but demyelinating peripheral neuropathy can affect all
    potentials of the blink reflex study. Motor neuron disorders such as amyotrophic lateral sclerosis
    do not typically affect the blink reflex.
60
Q
  1. A 55-year-old man presents with a 2-month history of progressive weakness. On examination he
    has mild proximal weakness in the upper and lower limbs. His muscle tone and bulk are normal
    and he has no facial weakness. Sensation is normal and deep tendon reflexes are 1+ and
    symmetrical. Which finding on electrodiagnostic testing is most consistent with this patient’s
    presentation?

(a) Prolonged or absent F waves
(b) Decreased recruitment ratio
(c) Motor unit potentials with amplitudes of 10 millivolts
(d) Normal number of phases of the motor unit potentials

A
  1. (b) The clinical presentation is most consistent with a myopathic picture. In myopathies the
    recruitment ratio is usually lower (
    amplitude potentials are associated with a neurogenic process. The number of phases of the motor
    units is usually increased in myopathy and in different forms and stages of neuropathies. F waves
    are typically prolonged or absent in neuropathies, but not in myopathies.
61
Q
  1. A 70-year-old man presents with a 3-month history of numbness in patchy areas over the limbs and
    torso. His numbness began in the left foot, then the right hand, followed by numbness over the
    back and all the limbs. He has no complaints of bowel or bladder problems. He has a long history
    of smoking. His examination reveals normal strength, normal cranial nerve function, but sensation
    is decreased to pin prick, vibration, and position in the limbs. Deep tendon reflexes are absent.
    Electrophysiologic studies show normal motor nerve conduction and needle examination of the
    upper and lower limb muscles. The sensory nerve conduction studies show small or absent
    responses. Based on this information what test would you order next?

(a) Nerve and muscle biopsies
(b) Radiologic studies to assess for a tumor
(c) Skin biopsy to assess small nerve fibers
(d) Repetitive nerve conduction studies

A
  1. (b) The clinical and electrophysiologic presentation is consistent with a sensory neuronopathy. With
    no evidence to suggest motor involvement, the numbness is likely a disorder of the dorsal root
    ganglion. There are only a few distinct disorders associated with acute or subacute cases described
    by the history and physical in this clinical vignette. They may be part of a paraneoplastic syndrome,
    connective tissue disorder such as Sjogren’s, a postinfectious condition, pyridoxine intoxication, or
    as an isolated autoimmune process. In this patient with a history of smoking, a cancer work up
    would include obtaining anatomic studies of the chest. Biopsies of the nerve, muscle, or skin would
    not add much to the case. Repetitive nerve conduction studies would be considered if a
    neuromuscular junction disorder was suspected.
62
Q
  1. What parameter of the motor unit action potential (MUAP) is the most sensitive to the distance
    between the generator source and recording electrode?

(a) Amplitude
(b) Phases
(c) Firing rate
(d) Duration

A
  1. (a) The amplitude is primarily influenced by the distance between the electrode’s recording surface and the electrical generator. The duration is a highly stable and reliable parameter of the motor unit.
    The other parameters are not affected by the distance.
63
Q

2008

  1. A 14-year-old girl is sent to you for electrodiagnostic evaluation of a left foot drop. The mother
    reports that the left foot drop has been present for about 2 months and that her daughter has lost
    about 40 pounds in the last 6 months. Nerve conduction studies and needle electromyography show
    the following data:

MOTOR NERVE CONDUCTION STUDIES
Nerve Stimulation Site Distal Latency (ms) Amplitude (mV) NCV
(m/s)
L Peroneal Ankle 4.2 1
Below knee 0.5 35
Above knee NR
R Peroneal Ankle 4.5 5.4
Below knee 5.0 47
Above knee 4.8 52
L Tibial Ankle 4.4 9
Knee 8.5 53

SENSORY NERVE CONDUCTION STUDIES
Nerve Stimulation Site Distal Latency (ms) Amplitude (µV)
L Sural Ankle - 14 cm 3.6 17
L Superficial peroneal Ankle - 12 cm NR
R Superficial peroneal Ankle - 12 cm 3.1 15

Needle Electromyography
Muscle Abnormal Recruitment
Spontaneous
Activity

L Vastus medialis 0 Normal
L Semimembranosus 0 Normal
L Short head of biceps
femoris 0 Normal
L Tibial anterior 3+ Reduced
L Medial gastrocnemius 0 None
L Peroneus longus 2+ Reduced
L Lumbar paraspinals 0 –
NCV = Nerve Conduction Velocity
NR = No response

The patient’s symptoms are most likely due to left

(a) peroneal nerve injury at the fibular head.
(b) deep peroneal nerve injury.
(c) sciatic nerve injury.
(d) lumbosacral plexopathy.

A
  1. (a) The electrophysiologic findings are consistent with common peroneal nerve compression at the fibular
    head. The normal study of the short head of the biceps femoris points to a lesion distal to the
    innervation of this muscle, and hence a lesion at or below the level of the knee. Lack of involvement of
    muscles innervated by other nerves points away from a plexopathy or sciatic nerve injury. Excessive
    weight loss can often be a factor in patients with peroneal nerve compression lesions.

Ref: Dumitru D, Zwarts MJ. Focal peripheral neuropathies. In: Dumitru D, Amato AA, Zwarts MJ,
editors. Electrodiagnostic medicine. 2nd ed. Philadelphia: Hanley & Belfus; 2002. p 1094-9.

64
Q
  1. What is the total duration of the motor unit action potential shown above?

(a) 8 ms
(b) 12 ms
(c) 18 ms
(d) 22 ms

A
  1. (c) The total duration of a motor unit is measured from the initial deflection from baseline to the final
    return to baseline.
65
Q
  1. While recording an antidromic sensory nerve action potential, you increase the distance between
    the active and reference electrode from 1cm to 4cm by moving the reference electrode. What is the
    effect on the onset latency and peak latency?Onset Latency Peak Latency

(a) No change Increase
(b) Increase No change
(c) Decrease Decrease
(d) Increase Increase

A
  1. (a) Increasing the interelectrode distance from 1 cm to 4 cm does not alter the onset latency, but increases
    the peak latency and amplitude of the sensory response. The onset latency does not change because the
    active electrode position is not changed. The sensory nerve action potential amplitude increases because
    less of the information is eliminated by differential amplification. Similarly, the peak latency also is
    prolonged as less of the signal is eliminated.

Ref: Dumitru D, Zwarts MJ. Instrumentation. In: Dumitru D, Amato AA, Zwarts MJ, editors.
Electrodiagnostic medicine. 2nd ed. Philadelphia: Hanley & Belfus; 2002. p 73.

66
Q
  1. A 65-year-old man is sent to you for electrodiagnostic assessment of his right facial droop. He
    presented to his internist 4 weeks ago with right facial pain and weakness, and decreased hearing.
    He was treated with high dose steroids without any improvement. His nerve conduction studies are
    as follows:

Nerve Distal Latency (ms) Amplitude (mV)
R Facial 3.5 0.8
L Facial 3.2 2.2

Blink Reflex Recording R1 (ms) R2 (ms)
R Trigeminal Right Absent Absent
Left – 33
L Trigeminal Right – Absent
Left 11.5 35
R1 – early component
R2 – late component

These findings are most consistent with right

(a) facial nerve lesion.
(b) trigeminal nerve lesion.
(c) pons lesion.
(d) lateral medulla lesion.

A
  1. (a) The blink reflex study has two responses, an early component (R1) and a late component (R2). The
    absence of the right R1 and R2 responses, but normal left R2 responses suggests a lesion affecting the
    right facial nerve. Additionally, the small right facial nerve response with direct stimulation suggests
    distal facial nerve degeneration. In a lesion involving the trigeminal nerve both R2 responses would be
    abnormal. In a pons or lateral medulla lesion the direct facial nerve response would be normal.
67
Q
  1. In patients with radiculopathy, how long does it take for morphologic changes of increased duration
    and amplitude to occur in the motor unit action potentials?

(a) 1 day
(b) 1 week
(c) 3 weeks
(d) 6 weeks

A
  1. (d) Reduced recruitment may be seen day 1. Positive waves may be seen in the paraspinals as early as 1
    week, and positive waves and fibrillations may be seen in the extremities by 3–5 weeks, but, because of
    reinnervation by axon sprouting, it takes at least 6 weeks to see increased amplitude and duration of
    muscle activity.

Ref: Dumitru, D, Zwarts, MJ. Radiculopathies. In Dumitru D, Amato AA, Zwarts, MJ, editors.
Electrodiagnostic medicine. 2nd ed. Philadelphia: Hanley & Belfus; 2002. p 732.

68
Q
  1. Following repair of a right distal biceps tendon rupture, a 31-year-old construction worker
    presents with problems extending his fingers. He had noticed swelling in the arm and forearm
    before his cast was removed about 4 weeks ago. He does not have any sensory complaints and the
    right superficial radial sensory nerve action potential is normal. Needle exam shows these data:

Muscle Abnormal Recruitment
Spontaneous
Activity
R Triceps 0 Normal
R Biceps 0 Normal
R Brachioradialis 0 Normal
R Extensor digitorum
communis 2+ Reduced
R Extensor indicis
proprius 2+ Reduced
R Flexor carpi ulnaris 0 Normal
R 1st dorsal
interosseous 0 Normal
R Extensor carpi
radialis 0 Normal
R Supraspinatus 0 Normal
R Cervical paraspinals 0 –

This patient most likely has a right

(a) radial mononeuropathy at the elbow.
(b) posterior interosseous neuropathy.
(c) posterior cord plexopathy.
(d) C7 and/or C8 radiculopathy.

A
  1. Answer: B
    Commentary: The electrophysiologic findings are consistent with involvement of the right
    posterior interosseous nerve (PIN). Typically in PIN injuries the triceps, brachioradialis, and
    extensor carpi radialis longus/brevis muscles are spared.
    Ref: Dumitru D, Zwarts MJ. Focal peripheral neuropathies. In: Dumitru D, Amato AA, Zwarts
    MJ, editors. Electrodiagnostic medicine. 2nd ed. Philadelphia: Hanley & Belfus; 2002. p 1092-3.
69
Q
  1. A 22-year-old woman with complaints of fluctuating weakness and abnormal fatigability that
    improves with rest is sent for electrophysiologic testing. Which electrophysiologic finding on
    routine testing would be most consistent with this clinical presentation?

(a) Small sensory nerve action potentials
(b) Slow motor nerve conduction velocities
(c) Variability in motor unit action potential amplitude
(d) Small compound muscle action potentials

A
  1. Answer: C
    Commentary: This patient’s presentation is most consistent with myasthenia gravis. The
    incidence of this condition is bimodal and affects women more than men in the younger age
    group. When one suspects myasthenia gravis the test of choice is repetitive stimulation.
    However, it is still important to assess for other possible problems and routine nerve conduction
    and needle electromyographic examination should be performed. The sensory component of the peripheral nervous system lacks a neuromuscular junction and hence the sensory responses
    should be normal. Motor amplitudes can be small, but this is usually only in severe cases. Motor
    conduction velocities are normal, since this study assesses the conduction along the motor fibers.
    Motor unit action potential amplitude variability is a characteristic abnormality observed during
    routine electromyography in patients with neuromuscular junction disorders. This finding is due
    to the variability in the total number of single muscle fibers being activated at any single time.
    Ref: Dumitru D, Amato AA. Neuromuscular junction disorders. In: Dumitru D, Amato AA,
    Zwarts MJ, editors. Electrodiagnostic medicine. 2nd ed. Philadelphia: Hanley & Belfus; 2002.p
    1149-50, 1153-4.
70
Q

68.

E-1 active electrode
E-2 reference electrode

What kind of needle electrode is depicted above?

(a) Monopolar
(b) Standard concentric
(c) Single fiber
(d) Bipolar concentric

A

68.Answer: C
Commentary: The single fiber electrode has an active electrode as a side port pickup and the
cannula serves as the reference. A separate ground electrode is also required.
Ref: Dumitru D, Zwarts MJ. Instrumentation. In: Dumitru D, Amato AA, Zwarts MJ, editors.
Electrodiagnostic medicine. 2nd ed. Philadelphia: Hanley & Belfus; 2002. p 74-6.

71
Q
  1. Which electrodiagnostic criterion is included in the diagnosis of peripheral nerve demyelination?

(a) Conduction velocity reduced in at least 4 nerves
(b) Compound muscle action potential conduction block in at least 3 nerves
(c) Prolonged distal motor latencies in at least 4 nerves
(d) Prolonged F-wave latency or absent F wave

A

88.Answer: D
Commentary: The criteria require conduction velocity to be reduced in 2 or more nerves,
compound muscle action potential conduction block or abnormal temporal dispersion in 1 or
more nerves, prolonged distal motor latencies in 2 or more nerves, and prolonged F wave or
absent F wave. Three of these four criteria must be present.
Ref: Amato AA, Dumitru D. Acquired neuropathies. In: Dumitru D, Amato AA, Zwarts MJ,
editors. Electrodiagnostic medicine. 2nd ed. Philadelphia: Hanley & Belfus; 2002. p 940.

72
Q
  1. Which electrodiagnostic finding is most consistent with neurogenic thoracic outlet syndrome?

(a) Small median motor response from the thenar muscles
(b) Abnormal response of the lateral antebrachial cutaneous nerve
(c) Abnormal median sensory responses
(d) Abnormal spontaneous activity in the pronator teres muscle

A

128.Answer: B
Commentary: Neurogenic thoracic outlet syndrome involves the lower trunk of the brachial
plexus; hence, sensory and motor loss develops in the C8–T1 distribution. Thumb abduction is
often affected. Sensory changes are usually in the distribution of the ulnar and medial
antebrachial cutaneous nerves.
Ref:Preston DC, Shapiro BE. Brachial plexopathy. In: Preston DC, Shapiro BE, editors.
Electromyography and neuromuscular disorders: clinical-electrophysiologic correlations.
Boston: Butterworth-Heinemann; 1998. p 438.

73
Q
  1. Which safety practice is the most appropriate when performing an electrodiagnostic study on a
    patient in a hospital bed?

(a) The device should be turned on after the placement of electrodes on the patient.
(b) An insulated extension cord should be used to connect the power line.
(c) More than 1 ground electrode should be attached to the patient.
(d) All electrical devices in contact with the patient should share a common ground.

A

8.Answer: (d)
Commentary: It is important to have all the electrical devices that are in contact with the patient
plugged into the same outlet to share a common ground. Similarly, only 1 ground electrode
should be used on the patient. To avoid power surges, the device should be turned on prior to the
application of any electrodes to the patient and turned off after the removal of electrodes.
Extension cords can increase leakage currents and should be avoided.

Ref: Dumitru D, Zwarts MJ. Instrumentation. In: Dumitru D, Amato AA, Zwarts MJ,
editors. Electrodiagnostic medicine. 2nd ed. Philadelphia: Hanley & Belfus; 2002. p 95-
6.

74
Q
  1. Which characteristic best describes fasciculation potentials?

(a) Semirhythmic in their firing pattern
(b) Morphologically the same as motor unit potentials
(c) Produced by ephaptic conduction between single muscle fibers
(d) Randomly firing single muscle fibers

A

28.Answer: (b)
Commentary: Fasciculation potentials are spontaneously firing motor unit potentials with the
same morphologic characteristics as that of a motor unit or polyphasic action potential. They
have an irregular firing pattern usually and the site of origin is unclear. Ephaptic conduction
between single muscle fibers is thought to be the mechanism for complex repetitive discharges.
Single muscle fiber potentials are much smaller and represent units such as a fibrillation potential.

Ref: Dumitru D, Zwarts MJ. Needle electromyography. In: Dumitru D, Amato AA,
Zwarts MJ, editors. Electrodiagnostic medicine. 2nd ed. Philadelphia: Hanley & Belfus;
2002. p 278-9.

75
Q
  1. Which modification can be used to increase the sensitivity of repetitive testing in neuromuscular
    junction disorders?

(a) Decrease the muscle temperature below 30°C.
(b) Allow the muscle to be rested for 6 minutes.
(c) Test the most distal muscles in the feet.
(d) Test the muscles after inducing ischemia.

A

38.Answer: (d)
Commentary: In postjunctional disorders, such as myasthenia gravis, the proximal muscles seem
to be more affected and it is thought that this difference is due to the increased temperature as one
gets closer to the core of the body. The higher temperatures potentiate a reduced safety factor.
Hence, the sensitivity of detecting a decrement response is higher with proximal muscle testing.
Maximal exercise can help in demonstrating postactivation exhaustion. When repetitive
stimulation is normal in both proximal and distal muscles, testing under ischemic conditions can
demonstrate a decrement in the responses.

Ref: Dumitru D, Amato AA. Neuromuscular junction disorders. In: Dumitru D, Amato
AA, Zwarts MJ, editors. Electrodiagnostic medicine. 2nd ed. Philadelphia: Hanley &
Belfus; 2002. p 1158-62

76
Q
  1. A 37-year-old man is sent to you for electrodiagnostic assessment for right lumbosacral
    radiculopathy. Nerve conduction studies of the right leg are normal. Needle exam shows the
    following:

NEEDLE ELECTROMYOGRAPHY

Muscle Abnormal Recruitment
Spontaneous
Activity
Adductor longus 0 Normal
Vastus medialis 0 Normal
Tensor fascia lata 1+ Normal
Semimembranosus 1+ Normal
Biceps femoris (short head) 0 Normal
Tibialis anterior 2+ Reduced
Medial gastrocnemius 0 Normal
Lumbar paraspinals 1+

Which root is most likely injured?

(a) L2
(b) L3
(c) L4
(d) L5

A

118.Answer: (d)
Commentary: The common root among the affected muscles is L5. It is often difficult to narrow
the involvement to a single root level.

77
Q
  1. Which muscle fiber is characterized by slow-twitch oxidative metabolic properties?

(a) Type 2a
(b) Type 1
(c) Type 2b
(d) Type 3

A
  1. Answer: (b)
    Commentary: There are 2 primary muscle fiber types in humans. They are categorized according
    to many different characteristics, including speed of contraction and sources of fuel. Type 1
    muscle fibers are slow-twitch with oxidative metabolic pathways. Type 2 muscle fibers are fast-
    twitch fibers. The type 2 fibers can then be further divided into fast-twitch glyclolytic, and fast-
    twitch oxidative glycolytic.

Ref: DeLateur BJ. Therapeutic exercise. In: Braddom RL, editor. Physical medicine and
rehabilitation. 2nd ed. Philadelphia: WB Saunders; 2000. p 398.

78
Q
  1. A 40-year-old man presents with a gradual onset of painful distal paresthesias while playing
    soccer. He feels clumsy and is falling more frequently. He has no known significant past medical
    history. Physical exam demonstrates normal symmetric strength throughout his upper and lower
    extremities, normal vibration sensation and normal Romberg testing. His electrodiagnostic exam
    results are as follows:
    MOTOR NERVE CONDUCTION STUDIES*
    Nerve Stimulation Site Distal Latency (ms) Amplitude (mV) NCV (m/s)
    R Fibular (Peroneal) Ankle 6.1 (≤ 6.5) 2.0 (≥2.0)
    Below knee 1.9 41 (≥40)
    Above knee 1.9 41
    L Fibular (Peroneal) Ankle 6.0 (≤ 6.5) 2.1 (≥2.0)
    Below knee 2.0 40 (≥40)
    R Tibial Ankle 6.1 (≤ 6.1) 3.0 (≥3.0)
    Knee 2.9 42 (≥40)
    R Median Wrist 4.4 (≤ 4.4) 5.2 (≥4.0)
    Elbow 5.2 49 (≥49)
    R Ulnar Wrist 3.5 (≤ 3.5) 7.0 (≥6.0)
    Below elbow 6.9 49 (≥49)
    * Normal values are in parentheses
    Abbreviation: NCV, nerve conduction velocity; R, right; L, left.

SENSORY NERVE CONDUCTION STUDIES*
Nerve Distal Latency (ms) Amplitude (µV)
R Sural NR NR
L Sural NR NR
R Median 3.9 (≤ 3.7) 12.0 (≥20.0)
R Ulnar 3.9 (≤ 3.5) 5.0 (≥15.0)
* Normal values are in parentheses
Abbreviations: NR, nonresponsive; R, right; L, left.

NEEDLE ELECTROMYOGRAPHY
Muscle Spontaneous Recruitment
Activity
R Gluteus medius 0 Normal
RVastus medialis 0 Normal
R Tibialis anterior 0 Normal
R Medial gastrocnemius 0 Normal
R Extensor hallicus longus 1+ Normal
R First dorsal interosseous (Pedis) 2+ Reduced
L First dorsal interosseous (Pedis) 2+ Reduced
R Pronator teres 0 Normal
R Abductor pollicus brevis 0 Normal
R First dorsal interosseous (Hand) 0 Normal
Abbreviations: R, right; L, left.

In addition, F waves were mildly prolonged in all motor nerves tested. His history and exam is
most consistent with a diagnosis of sensorimotor neuropathy due to

(a) diabetes or glucose intolerance.
(b) uremic disease.
(c) chronic inflammatory demyelinating polyradiculopathy.
(d) hereditary motor sensory neuropathy

A

8.Answer: (a)
Commentary: Diabetic neuropathy electrophysiological exam may demonstrate sensory nerve
conduction study abnormalities with normal motor nerve conduction studies but positive
fibrillation potentials distally on needle examination. In this case, motor nerve conduction studies
are borderline normal. It is likely that had F waves been obtained they would be mildly
prolonged. A neuropathy due to uremic disease would likely demonstrate pronounced conduction
slowing and low amplitude responses in sensory and motor nerves, and likely would not be
undiagnosed prior to the presentation described in the scenario. Chronic inflammatory
demyelinating polyradiculopathy (CIDP) and hereditary motor sensory neuropathy (HMSN-1)
represent demyelinating processes, and there is no evidence of this finding in the results.

Reference: (a) Albers JW. Evaluation of the patient with suspected peripheral neuropathy. In:
Pease WS, Lew HL, Johnson EW, editors. Johnson’s practical electromyography, 4th ed.
Philadelphia: Lippincott Williams & Wilkins; 2007. p 308-27.
(b) Preston DC, Shapiro BE. Polyneuropathy. In: Preston DC, Shapiro BE, editors.
Electromyography and neuromuscular disorders. 2nd ed. Philadelphia: Butterworth-Heinemann;
2005. p 389-420.

79
Q
  1. A patient having difficulty late in the day getting up from a chair, going up or down stairs, and
    reaching with his arms presents for electrodiagnostic studies. Physical exam demonstrates normal
    deep tendon reflexes and normal findings on manual muscle testing. Standard sensory and motor
    nerve conduction studies are normal. Repetitive axillary nerve stimulation (RNS) performed at
    2Hz demonstrates 20% decremental response. Immediately after exercise, the RNS decrement is
    no longer observed. Three minutes following exercise, however, the decrement is greater. Needle
    electromyography results are normal.

Upon further investigation, you would most likely find what additional clinical finding?

(a) Asthma
(b) Dry mouth
(c) Ptosis
(d) Skin rash

A
  1. Answer (c)
    Commentary: The patient presents with myasthenia gravis (MG), a postsynaptic neuromuscular
    junction disorder. Ptosis and extraocular weakness often occur in MG. Lambert-Eaton myasthenic
    syndrome (LEMS), a presynaptic neuromuscular junction disorder, would demonstrate
    postexercise facilitation (at least 100% increase in first response CMAP immediately following
    exercise) and likely have low-amplitude baseline CMAP results. Autonomic symptoms such as
    dry mouth often accompany LEMS. Long-term steroid treatment for asthma may cause myopathy
    without significant needle EMG results, but RNS would be normal. Although dermatomyositis
    typically presents with proximal weakness, no abnormalities characteristic of an inflammatory
    myopathy were seen on needle electromyography.

Reference: Preston DC, Shapiro BE. Neuromuscular junction disorders. In: Preston DC, Shapiro
BE, editors. Electromyography and neuromuscular disorders, 2nd ed. Philadelphia: Butterworth-
Heinemann; 2005. p 553-61.

80
Q
  1. Which type of study best differentiates a severe polyradiculopathy from amyotrophic lateral
    sclerosis (ALS)?

(a) Motor nerve conduction studies of upper and lower extremities
(b) Needle electromyography of thoracic paraspinals or bulbar muscles
(c) Sensory nerve conduction studies of upper and lower extremities
(d) Needle electromyography of multiple extremities

A

48.Answer (b)
Commentary: Sensory nerve conduction studies are normal in both radiculopathy and motor
neuron disease. Motor nerve conduction studies are also often normal in both diseases. Both
diseases may demonstrate abnormal needle examination in multiple extremities. Thoracic
paraspinals and bulbar muscle examinations are most helpful in differentiating severe
polyradiculopathy from amyotrophic lateral sclerosis (ALS), since one would expect these studies
to be normal in radiculopathy but may be abnormal in ALS.
Reference: Preston DC, Shapiro BE. Amyotrophic lateral sclerosis and its variants. In: Preston
DC, Shapiro BE, editors. Electromyography and neuromuscular disorders, 2nd ed. Philadelphia:
Butterworth-Heinemann; 2005. p 428-32.

81
Q
  1. In a patient with a neuromuscular junction disorder, which electrodiagnostic results for compound
    muscle action potential (CMAP), motor unit action potential (MUAP) or nerve action potential
    (SNAP) may be misleading if the limb is cold?

(a) Diminished CMAP decrement on repetitive nerve stimulation
(b) Diminished polyphasia of the MUAP
(c) Shortened distal latency of the CMAP
(d) Decreased amplitude of the SNAP

A
  1. Answer (a)
    Commentary: In neuromuscular junction (NMJ) disorders, compound muscle action potential
    (CMAP) decrement may be diminished if the limb is cold, likely due to decreased functioning of
    acetylcholinesterase. Cool temperatures may alter results by slowing nerve conduction velocity,
    prolonging distal latency, increasing amplitude and duration of sensory nerve action potential
    (SNAP) and CMAP and motor unit action potential (MUAP), increasing phases of MUAP.

Reference: Preston DC, Shapiro BE. In: Preston DC, Shapiro BE, editors. Electromyography
and neuromuscular disorders, 2nd ed. Philadelphia: Butterworth-Heinemann; 2005. p 71, 87-9.

82
Q
  1. When performing needle electromyography in someone therapeutically anticoagulated with
    warfarin, which muscle would be the safest to examine?

(a) Tibialis posterior
(b) Paraspinal muscles
(c) Deltoid
(d) Flexor pollicis longus

A

118.Answer (c)
Commentary: Although needle EMG is generally safe in patients taking warfarin, whenever
bleeding risk is increased it is best to limit the needle examination to a few superficial muscles,
where prolonged compression can be performed if necessary. The pronator teres and tibialis
posterior are deep muscles in which, theoretically, a hematoma could develop. It is also best to
avoid needle insertion into areas where possible hematoma could compress a nerve or artery, such
as exiting spinal nerves near paraspinals or the radial artery near the flexor pollicis longus.
Reference: Preston DC, Shapiro BE. Electrical safety and iatrogenic complications of
electrodiagnostic studies. In: Preston DC, Shapiro BE, editors. Electromyography and
neuromuscular disorders: clinical-electrophysiologic correlations, 2nd ed. Philadelphia:
Butterworth-Heinemann; 2005. p 658-9.

83
Q
  1. A man presents to your clinic complaining of buttock pain that radiates posteriorly down the
    thigh. On exam you note that he has a leg length discrepancy, symptoms are provoked by placing
    the affected limb in the FAIR position (hip in flexion, adduction and internal rotation), and he has
    a positive straight leg raise test. He has normal nerve conduction studies and a normal needle
    electromyography test. Treatments that may be beneficial include

(a) stretching exercises of the iliotibial band and corticosteroid injection of the greater
trochanteric bursae.
(b) a lumbar stabilization exercise program and coricosteroid lumbar epidural spinal
injection.
(c) a lumbar stabilization exercise program and botulinum toxin injection of the lumbar
paraspinals.
(d) stretching exercises in the FAIR position and botulinum toxin injection to the piriformis.

A
  1. Answer (d)
    Commentary: This is a description of piriformis syndrome. Although some positive findings on
    needle examination may be seen with piriformis syndrome, electrodiagnostic studies are often
    normal. On the other hand, positive findings are expected in cases of lumbar radiculopathy.
    Conservative treatment of piriformis syndrome begins with piriformis stretching (FAIR position
    is a good position for this) and nonsteroidal anti-inflammatory drugs (NSAIDs), followed by
    lumbosacral stabilization, hip strengthening, and myofascial release. Botulinum toxin relieves
    pain via multiple mechanisms and is increasingly used in the treatment of myofascial dysfunction.
    A lumbar stabilization exercise program and botulinum toxin injection of the lumbar paraspinals
    may help relieve some of this patient’s pain if he also has low back pain, but would not address
    the main issue, piriformis syndrome. Stretching exercises of the iliotibial band and corticosteroid
    injection of the greater trochanteric bursae would be the treatment for greater trochanteric
    bursitis. A lumbar stabilization exercise program and corticosteroid lumbar epidural spinal
    injection would treat a lumbar radiculopathy.
    Reference: Kirschner JS, Foye PM, Cole JL. Piriformis syndrome, diagnosis and treatmtent.
    Muscle Nerve 2009;40:10-8.
84
Q
  1. What is the difference between median mononeuropathy at the wrist and CTS? (605)
A

Median mononeuropathy at the wrist is an electrodiagnostic finding and CTS is a clinical one.

85
Q
  1. Why are sensory fibers more susceptible to ischemic damage than motor?
A

) Sensory fibers have a larger proportion of large myelinated fibers, which have a higher energy requirement and thus are more susceptible to ischemic damage.

86
Q

Your patient has CTS symptoms only in the right hand. Rigorous NCS on the right are normal. Do you need to test the left hand?

A

No. If initial comparison NCS on the symptomatic side are normal, further NCSs are rarely needed. However were symptoms bilateral, then NCS on the opposite side would be useful.

87
Q
  1. Your patient with CTS symptoms has low median SNAP amplitude but normal latency. Does this finding allow you to make a diagnosis?
A

) Not by itself. SNAP amplitude has not been reported as reliable in the diagnosis of CTS. In the absence of a prolonged distal latency, a low SNAP amplitude has limited localizing value. If you do a palmar stimulation for the same SNAP, you may be able to demonstrate conduction block across the carpal tunnel, which would be helpful for diagnosis.

88
Q

Your patient has prolonged SNAP latency of only one median digit. Name a pathology other than CTS that could cause this. (598)

A

“A prolonged finger to wrist latency can also be the result of a focal lesion distal to the carpal tunnel, such as a digitial neuropathy.” I would like to know how common digital neuropathies are. Should we discard prolonged comparative latencies confined to one digit?

89
Q

You obtain SNAP latencies to digit III stimulating at wrist and palm. How would findings differ in CTS vs PPN?

A

In CTS the slower segment would be the proximal one that crosses the carpal tunnel. In PPN the slower segment would be the distal one.

90
Q
  1. You obtain SNAPs to digit III stimulating at wrist and palm. What decrement in amplitude suggests conduction block?
A

Determining the exact criteria for conduction block in sensory studies is difficult, but an increase in the palm SNAP amplitude of at least 50% compared with wrist stimulation has been reported to suggest conduction block. This means that if you get an amplitude of 10 at the wrist, amplitude at the palm of 15 might suggest conduction block. Preston (p142) sets the threshold at 60%. In other words, wrist amplitude of 10 and palm amplitude of 16.

91
Q
  1. You obtain the transcarpal median mixed nerve potential recorded at the wrist. Which contributes more to the waveform: motor or sensory fibers?
A

Sensory fibers contribute more. Median palmar stimulation evokes a mixed nerve action potential from motor fibers innervating the second lumbrical and sensory digital afferents from the index and middle fingers. However, the potential at the wrist is generated primarily by sensory fibers. (Sensory fibers have a larger proportion of large myelinated fibers, which have a higher energy requirement and thus are more susceptible to ischemic damage.)

92
Q
  1. For transcarpal mixed nerve potentials, is shock artifact more likely to be a problem for median or ulnar potential? (599)
A

Shock artifact is more likely to cause difficulty obtaining a good response of ulnar mixed nerve study. Because the distance between stimulation and recording electrode is relatively small, shock artifact is a potential problem for both median and ulnar studies, but may be more of a problem for ulnar.

93
Q
  1. You compare the distal latencies for SNAPs to digits 2 and 5. What delta is significant for the general population? (600)
A

Delta of at least 0.5ms is significant between distal latencies at the wrist for SNAPs to digits 2 and 5 in the general population. This threshold of 0.5ms being abnormal is the same delta seen for comparison studies to digit IV.

94
Q

You compare the distal latencies for SNAPs to digits 2 and 5. If this patient is in the general population the delta is abnormal if it is 0.5ms or greater. However your patient is an “active worker.” What delta must they have to be abnormal?

A

(600) At least 0.8ms. For active workers, the delta between SNAPs to digits 2 and 5 must be at least 0.8 ms to be abnormal.

95
Q
  1. You compare the distal latencies for SNAPs to digits 2 and 5. If this patient is in the general population the delta is abnormal if it is 0.5ms or greater. For active workers it is 0.8ms or greater. However your patient is a mild diabetic. What delta must they have to be abnormal? (600)
A

At least 1.0 ms. For mild diabetics, the delta between SNAPs to digits 2 and 5 must be at least 1.0 ms to be abnormal. This is twice the delta needed to be abnormal in the general population.

96
Q
  1. You do the digit I comparison test. You know that the results will be altered by the thumb’s position during measurements. What position should you put the thumb in to standardize measurements? (600)
A

The text says that the thumb should be extended, but the cartoon shows it abducted. Abduct the thumb to standardize digit I comparison test measurements.

97
Q
  1. What delta is abnormal for digit I comparison test? (600)
A

Digit I may be abnormal with delta of 0.5 or 0.4 depending on which studies you go by. Preston avoids giving a value and doesn’t demonstrate the digit I test in their NCS section. Preston comments that measurement errors are a drawback to the digit I test.

98
Q
  1. The cartoon shows measurements between ring electrodes and stimulation sites for comparison test to digit I. Which of the two stim sites involves a bent path? (601)
A

In the cartoon, both median and radial nerves are measured along bent paths. The median nerve travels in a straight line from ring electrode to carpal tunnel, where it pivots to run parallel to the forearm. The radial nerve travels along the radial aspect of the abducted thumb and pivots at the wrist to run parallel to the forearm.

99
Q
  1. You set up an antidromic SNAP to digit III and perform centimetering studies from wrist to palm. How much change in latency over 1 cm is abnormally large? (600)
A

For SNAP centimetering studies, a latency change of at least 0.5ms is abnormally large. This threshold of at least 0.5ms is also the most common abnormal value for comparison studies. Latency changes of 0.5 ms across a centimeter is abnormal.

100
Q
  1. Does the monograph believe that centimetering is a reliable method? (600)
A

The monograph does state that inching is reliable, but then proceeds to list the many potential pitfalls that can make it unreliable including cathodal migration, susceptibility to measurement error and potential interference of stimulus artifact.

101
Q
  1. In your patient with possible CTS, you calculate the CSI. That is to say, you add the deltas of comparison tests of digit IV, digit I and mixed nerve transcarpal. What value is abnormal? (600)
A

) The CSI is abnormal if it is at least 0.9ms. This has a sensitivity of 83% and specificity of 95%.

102
Q
  1. You perform the first two comparison tests for the CSI in a patient with suspected CTS. These tests are both definitely abnormal. Do you need to perform the third test? (597)
A

No. Two comparison techniques that clearly agree – either normal or abnormal – should be adequate to confirm the diagnosis. In cases where the testing is borderline, extra testing and/or the CSI can help clarify.

103
Q

Your patient has CTS symptoms only in the right hand. CSI is positive. Do you need to test the left hand with NCS? (597)

A

No. When comparison NCS are abnormal for CTS in one limb and that limb is the only symptomatic limb, the AANEM guidelines do not recommend performing NCS on the opposite hand.

104
Q
  1. . For your patient with likely CTS, the median SNAP to digit IV is 0.7 ms slower than the ulnar. Do you need to do the other two tests to complete the CSI? (601)
A

) The threshold delta to be abnormal is 0.5ms. Studies have shown that if the first of any of the three CSI components are significantly abnormal, the CSI is 98% likely to be abnormal, so you don’t have to complete the CSI. However, Preston cautions never to base your diagnosis on just one test, and I would prefer to get a second comparison test that is consistent.

105
Q
  1. Your patient is worried about a false positive CSI. You tell them that if the CSI is at least 0.9ms that is 95% specific. They want the test to be even more specific. What CSI delta should you use to get specificity to almost 100%? (600)
A

One study found that CSI threshold of 1.1 is nearly 100% specific but lowers sensitivity from 83% to 82%.

106
Q
  1. The AANEM published practice guidelines giving sensitivity and specificity of various tests for CTS. Also, this 2011 monograph mentions sensitivity and specificity values for some tests. Should we memorize these values? (603)
A

Apparently not. “A consensus conference employed to refine the epidemiologic case definition of CTS concluded that there is no gold standard for the diagnosis and therefore attempts at defining sensitivity and specificity are estimates at best.”

107
Q
  1. The monograph authors admit that they are guessing when they ascribe a sensitivity to electrodiagnostics for CTS. What sensitivity do they say NCS have for CTS? (604)
A

“approximately 10-15% of subjects with clinical CTS will have normal NCS, which reflects a sensitivity of 85-90%.” As does Preston, these authors conclude that a combination of electrodiagnostic and clinical findings should be used to establish the diagnosis. They say that electrodiagnosis is 85-90% sensitive for CTS but admit this is a guess.

108
Q
  1. Your patient has a CSI of 3. They ask if they are likely to benefit from surgery. What do you tell them? (601)
A

A retrospective trial found that somewhat better surgical outcomes were found in patients with CSI between 2.5 and 4.6.

109
Q
  1. CSI between 2.5 and 4.6 may be the most likely to benefit from surgery. What delta between ulnar and median CMAP distal latency is considered abnormal by the original AANEM minimonograph on CTS? (see question 31 for those study questions or page 1477 of that text)
A

Delta of 1.8ms is abnormal. If median CMAP latency is 4.0 and ulnar is 2.2, that delta of 1.8ms suggests moderate CTS to the author.

110
Q
  1. CSI between 2.5 and 4.6 may be the most likely to benefit from surgery. What does the 2011 monograph on CTS say is a significant delta between ulnar and median CMAP distal latency? (601)
A

They give a range of 1.2 to 1.8 ms as the minimum significant delta. The original minimonograph uses 1.8 ms as the minimum significant delta.

111
Q
  1. CSI between which values are most likely to benefit from surgery? (601)
A

CSI between 2.5 and 4.6 may be the most likely to benefit from surgery.

112
Q
  1. For comparison of distal CMAP latency of second lumbrical vs interossei, both Preston and the monograph can be thought of as agreeing that our threshold for abnormal is at least 0.5ms delta. Does the monograph feel that this is a specific test? (603)
A

The monograph sites a study that cautions that the 0.5ms threshold may result in false positives.

113
Q
  1. To look for conduction block in the carpal tunnel, you obtain APB CMAP stimulating at palm and wrist. Give two reasons why the palm stimulation may produce a spuriously large amplitude. (603)
A

Large amplitudes may erroneously result with palm stimulation because of direct stimulation of the APB or because of costimulation of the deep motor branch of the ulnar nerve which may activate ulnar-innervated thenar muscles. Because of these two reasons, the monograph says that this technique must be used with caution and with an eye for any changes in CMAP morphology. Preston cautions that costimulation in any location may not result in noticeable change in CMAP morphology, and the monograph warns against false-positive conduction block with this technique.

114
Q
  1. What study do the authors recommend to show conduction block across the carpal tunnel? (605)
A

They suggest SNAP at wrist and midpalm. CMAP at midpalm may be unreliable because of direct stimulation of the muscle or of co-stimulation of deep palmar branch of the ulnar nerve.

115
Q
  1. Characterize the impact of Martin-Gruber anastomosis on SNAPs. (603)
A

MGA never effects sensory fibers. It is a purely motor anastomosis.

116
Q
  1. Your patient is referred to assess for possible CTS. They have significantly delayed distal latency to the APB with wrist stimulation but the CSI comparison tests are all normal. At this point in the study, what can you say about diagnosis of CTS? (603)
A

) The monograph authors have found no published cases of CTS in which motor latency is abnormal and the comparison sensitive sensory studies are normal.

117
Q
  1. Your patient has delayed distal latency to the APB with wrist stimulation but the CSI comparison tests are all normal. Name a neuropathy distal to the elbow that could cause this. (603)
A

There have been reported cases of recurrent branch median neuropathy at the wrist. The recurrent branch of the median nerve may also be called the ‘million dollar nerve.’ It supplies only the thenar muscles. Other branches of median nerve in the hand supply the lumbricals and sensation.

118
Q
  1. Your patient has delayed distal latency to the APB with wrist stimulation but the CSI comparison tests are all normal. One explanation could be recurrent branch median neuropathy at the wrist. Name three other potential causes of this isolated motor nerve abnormality. (603)
A

Three other causes include C8 and/or T1 radiculopathy, anterior horn cell disease, and multifocal motor neuropathy with conduction block. Recall that MMNCB is true to its name in sparing sensory fibers and that testing SNAPs for conduction block over the same segment helps confirm diagnosis.

119
Q
  1. According to the monograph, which comparison test is the best for work-up of median mononeuropathy at the wrist? (603)
A

There isn’t enough evidence to say that any given comparison test is better than another. However there is good evidence that comparison tests are better than measuring standard tests against published values.

120
Q
  1. The monograph authors guestimate that 10-15% of patients with clinical CTS will have normal EDX. What is their explanation for the symptoms in patients with normal NCS? (604)
A

They suggest that intermittent symptoms may be due to reversible ischemia with reversible conduction block. However, the authors also say that provactive EDX tests are not good tests because of false positives.

121
Q
  1. Although the authors admit that specificity can only be guestimated, they do take a position on the specificity of EDX testing for CTS. Is EDX testing qualitatively specific for CTS? (604)
A

They say that the sensitivity is 85-90% and the specificity is 82-85% in certain populations. So they don’t think that EDX are especially specific for CTS.

122
Q
  1. The authors say that EDX specificity is 82-85% in certain populations. Which population do they suggest may be especially vulnerable to false positives? (604)
A

They suggest that workers with hand-intensive jobs with vague hand symptoms may be especially vulnerable to false positive EDX testing.

123
Q
  1. Your patient works a hand-intensive job but has no hand symptoms. Routine median SNAP latency to digit II is at least 0.5ms greater than ulnar SNAP latency to digit V. Does this person have an increased risk of some day having CTS? (604)
A

Studies found no higher risk of CTS 2 years later but did find a small but increased risk of CTS at 6 and 11 years later compared to controls who had no normal latency deltas. At least 75% of these patients did not have CTS at 6-11 years follow-up. The monograph authors conclude that asymptomatic patients with SNAP latency delta of at least 0.5ms may be at increased risk for developing CTS later.

124
Q
  1. Your patient is a mild diabetic and has no symptoms of CTS. Their CSI is 0.9. Should you be cautious about diagnosing median mononeuropathy at the wrist? (604)
A

Yes. Among mild diabetics without CTS symptoms, a large multicenter trial showed that delta of 1.0ms was the 2 standard deviation upper limit of normal for comparing SNAP latencies to digits 2 and 5. Using 0.5ms as your cutoff would have resulted in many false positives. The diagnosis of CTS in diabetics is more difficult, and values obtained from normal populations can be misleading. The authors of that study suggest using larger deltas, such as 1.0ms, for comparison tests for diabetics.

125
Q
  1. Your patient works in a hand-intensive job such as assembly or clerical work. What delta is abnormal for comparing SNAP latencies to digits 2 and 5? (604)
A

) For hand intensive workers, 0.8ms may be a threshold that gives you greater specificity. A study of more than 300 such workers found that 0.8ms was the upper limit of normal for median to ulnar comparison SNAPs to digit II and V. The monograph authors point out that these 300 data points represents more rigor than is used for most normal value studies.

126
Q
  1. For SNAP comparisons to digits II and V, diabetics are normal with deltas less than 1.0ms and workers at hand-intensive jobs are normal with deltas less than 0.8ms. Do the authors generalize these results to urge use of higher delta thresholds for any patient with neuropathy? (605)
A

Yes. Unfortunately, firm guidelines with exact threshold for NCS tests are not available for neuropathy patients. The authors point to the aforementioned studies to urge caution in those populations.

127
Q
  1. The monograph authors guestimate that for CTS, EDX sensitivity is 85-90% and the specificity is 82-85% in certain populations. Your patient had surgery for CTS with resolution of symptoms. Repeat NCS one year later still are consistent with CTS. What do you conclude? (605)
A

Slowed but improved latencies may persist after surgery, even if symptoms are resolved.

128
Q
  1. The monograph authors guestimate that for CTS, EDX sensitivity is 85-90% and the specificity is 82-85% in certain populations. Your patient has CTS symptoms on the right and CSI is positive. Should you perform needle study? (598)
A

) According to the AANEM practice recommendations, needle study is optional. If you perform it, you should needle a thenar muscle supplied by the median nerve in the affected limb and you could consider sampling other muscles supplied by C5 to T1. Needle study is optional in electrodiagnosis of CTS.

129
Q
  1. NCS and clinical picture are convincing for diagnosis of CTS. What is a reason to do needle study? (605)
A

Needle study is listed as optional by the AANEM practice parameter. One reason to do it is to document axonal loss. However, if documenting axonal loss won’t affect management, you needn’t do the needle study.

130
Q
  1. One reason to do needle study in work-up of CTS is to evaluate for axonal loss. The monograph authors suggest that patients with weakness or atrophy of thenar muscles or reduced CMAP amplitude would benefit from needle study to document axonal loss. (I suggest that only if these findings are equivocal would needle study be necessary to confirm axonal loss.) What’s another reason to do needle study in work-up of CTS? (605)
A

Needle study can help rule out other causes of the clinical picture such as radiculopathy, proximal median neuropathy or PPN.

131
Q
  1. Should you classify CTS as mild, moderate or severe in your report? (605)
A

The use of these classification scales has been discouraged for several reasons including the fact that they aren’t based on any evidence and correlate poorly with clinical severity. Both this monograph and the AANEM practice guidelines provide a classification system and also tell you not to base your classification solely on EDX.

132
Q
  1. What EDX findings define mild CTS? (605)
A

Mild is prolonged SNAP latencies, either absolute or relative.

133
Q
  1. What EDX findings define moderate CTS? (605)
A

Moderate is prolonged SNAP and CMAP latencies, either absolute or relative.

134
Q
  1. What EDX findings define severe CTS? (605)
A

Severe is evidence of axonal loss.

135
Q
  1. You have now memorized this classification system for CTS severity. Should you use it? (605)
A

Probably not. However knowing the system will at least help you understand what other electromyographers may mean in their reports.

136
Q
  1. How can you demonstrate axonal loss using NCS? (605)
A

You can demonstrate axonal loss with NCS by showing reduced amplitudes and by ruling out conduction block across the carpal tunnel.

137
Q
  1. What study do the authors recommend to show conduction block across the carpal tunnel? (605)
A

They suggest SNAP at wrist and midpalm. CMAP at midpalm may be unreliable because of direct stimulation of the muscle or of co-stimulation of deep palmar branch of the ulnar nerve.

138
Q
  1. What ratio of palm to wrist SNAP amplitude suggests conduction block? (599, Preston 142)
A

Preston says palm to wrist ratio of 1.6; this monograph says 1.5.

139
Q
  1. You notice reduced amplitudes of median SNAPs and suspect axonal loss. However midpalm SNAP demonstrates conduction block. So now you aren’t sure if there was axonal loss. What’s another technique besides NCS that can demonstrate axonal loss in CTS? (605)
A

) You can demonstrate axonal loss with needle study.

140
Q
  1. With EDX, you demonstrate CTS with median axonal loss. Your patient wants to skip conservative treatment and get surgery. What do you say? (605)
A

AAOS guidelines say that early surgery without conservative treatment is an option in patients with axon loss. In preferring surgery to conservative treatment, this patient is not departing from the realm of standard care.

141
Q
  1. Your patient is slender. Are they more or less likely to have asymptomatic median mononeuropathy at the wrist than an obese person? (605)
A

Obese people are more likely to have asymptomatic MMW. Several studies have confirmed this. You should therefore be cautious about diagnosing CTS in an obese patient with positive EDX but a vague clinical picture.

142
Q

Your patient has classic CTS symptoms but EDX are negative. Do the monograph authors believe that he likely has CTS? (605)

A

Yes. “On occasion the EDX test is normal in patients with classical CTS symptoms. In these cases, the test is likely a false negative and the patient still has CTS. This can be explained in the report.” I am impressed that the authors are so candid about the limitations of EDX!

143
Q
  1. The authors believe that a classic clinical case of CTS is likely CTS regardless of the EDX results. This begs the question of the utility of EDX. Do the authors believe that EDX is useful in CTS evaluation? If so, what is that role? (605)
A

The authors do believe that EDX is useful in CTS evaluation. They see EDX as useful when the clinical picture is less clear.

144
Q
A
145
Q
A
146
Q
A